Breast Reconstruction Flashcards

1
Q

An otherwise healthy 52-year-old woman is scheduled to undergo bilateral breast reconstruction with abdominal perforator flaps. She does not smoke cigarettes. Which of the following is the most appropriate antibiotic prophylaxis regimen?
A) Preoperative antibiotics
B) Preoperative chlorhexidine bath
C) Preoperative and intraoperative antibiotics
D) Preoperative, intraoperative, and postoperative antibiotics for 7 days
E) Preoperative, intraoperative, and postoperative antibiotics until all drains are removed

A

C) Preoperative and intraoperative antibiotics

A single dose of preoperative antibiotic intravenously is appropriate for short procedures; however, therapeutic levels of antibiotic should be maintained throughout the lengthy surgery described.

Continued administration of prophylactic antibiotics for 7 days following surgery or until drains are removed has not been proven to reduce the incidence of surgical site infections. This regimen also promotes drug resistance and adverse patient reactions.

How well did you know this?
1
Not at all
2
3
4
5
Perfectly
2
Q

For a healthy patient undergoing a lengthy autologous breast reconstruction, the most appropriate regimen of antibiotic prophylaxis consists of:

A

For a healthy patient undergoing a lengthy autologous breast reconstruction, the most appropriate regimen of antibiotic prophylaxis consists of a preoperative dose of intravenous antibiotic and intraoperative antibiotics to maintain therapeutic levels during operation and, at most, a few hours after closure

How well did you know this?
1
Not at all
2
3
4
5
Perfectly
3
Q
A 52-year-old woman with breast cancer undergoes right mastectomy and reconstruction with a free transverse rectus abdominis musculocutaneous (TRAM) flap. The procedure is uneventful. In the recovery room, the patient’s husband says that she has been smoking one pack of cigarettes daily up to the day of surgery. Which of the following postoperative complications is most likely to occur?
A) Flap hematoma
B) Mastectomy skin loss
C) Microvascular thrombosis
D) Partial flap loss
E) Superficial infection
A

B) Mastectomy skin loss

How well did you know this?
1
Not at all
2
3
4
5
Perfectly
4
Q

Smoking vs risks that have been increased vs unchanged

A

Patients who smoke cigarettes and who undergo breast reconstruction with a free flap have a higher rate of mastectomy skin loss. In addition, they havea high rate of donor site abdominal flap necrosis, umbilical necrosis, and hernias. There is no increase in microsurgical complications, flap-related complications (partial flap loss or fat necrosis), infections, or hematomas

How well did you know this?
1
Not at all
2
3
4
5
Perfectly
5
Q

Patients should stop smoking how long before breast reconstruction?

A

Recommendations are for patients to stop smoking at least 4 weeks prior to breast reconstruction. Patients who quit smoking prior to this period have a lower risk of perioperative complications when compared to active smokers

How well did you know this?
1
Not at all
2
3
4
5
Perfectly
6
Q

A 53-year-old woman is referred for consultation regarding breast reconstruction following mastectomy. Autologous breast reconstruction options are discussed. Which of the following is an advantage of the pedicled transverse rectus abdominis musculocutaneous (TRAM) flap over the free TRAM flap?
A) Decreased complications in patients with a history of smoking
B) Decreased incidence of complete flap loss
C) Decreased risk of abdominal hernia
D) Decreased risk of fat necrosis
E) Ease of insetting and flap shaping

A

B) Decreased incidence of complete flap loss

How well did you know this?
1
Not at all
2
3
4
5
Perfectly
7
Q

Pros/Cons of peddled TRAM

A

The pedicled TRAM flap is characterized by a shorter operative time and a decreased risk of complete flap loss when compared to the free flaps. However, the pedicled TRAM flap shows a higher incidence of fat necrosis with partial flap loss, as well as an increased length of stay.

How well did you know this?
1
Not at all
2
3
4
5
Perfectly
8
Q

Benefits of free flaps over pedicled for breast reconstruction

A

Free flaps offer ease of flap shaping and insetting, as well as a decreased risk of abdominal hernia and abdominal wall weakness. Free flaps are more suitable for patients who are diabetic, are overweight, or smoke cigarettes.

How well did you know this?
1
Not at all
2
3
4
5
Perfectly
9
Q

A 45-year-old woman with a T2 N0 M0 invasive ductal carcinoma in the inferior pole of the left breast is scheduled to undergo segmental mastectomy followed by radiation therapy. She currently wears a size 36E brassiere and wants to have any cup size from a C to D. After segmental resection of the tumor, which of the following procedures is most likely to yield the best cosmetic result in this patient?
A) Bilateral reduction mammaplasty
B) Primary closure of the left breast and reduction mammaplasty of the right breast
C) Reconstruction of the left breast with a latissimus dorsi musculocutaneous flap and reduction mammaplasty of the right breast
D) Reconstruction of the left breast with a transverse rectus abdominis musculocutaneous flap and mastopexy of the right breast
E) A single-stage breast reconstruction with placement of a silicone prosthesis in the left breast and reduction mammaplasty of the right breast

A

A) Bilateral reduction mammaplasty

In a patient with large breasts, in whom a partial mastectomy is required, reduction mammaplasty is an appropriate management. This procedure will potentially relieve symptoms of macromastia, reduce the amount of breast tissue present in both breasts, and offer the best aesthetic outcome.

How well did you know this?
1
Not at all
2
3
4
5
Perfectly
10
Q

A 46-year-old woman with ductal carcinoma in situ is scheduled to undergo right mastectomy. Immediate reconstruction with a tissue expander and acellular dermal matrix (ADM) is planned. Which of the following is an expected outcome with use of ADM?
A) Decreased formation of seroma
B) Decreased incidence of hematoma
C) Decreased risk of infection
D) Increased capsule contracture
E) Increased initial fill of the expander

A

E) Increased initial fill of the expander

How well did you know this?
1
Not at all
2
3
4
5
Perfectly
11
Q

ADM and initial fill of saline for an expander

A

According to Sbitany, et al, acellular dermal matrix (ADM) allows for a greater initial fill of saline. This potentially improves cosmetic outcome, as it better capitalizes on preserved mastectomy skin for reconstruction. Sbitany, et al, concluded that ADM-assisted prosthesis breast reconstruction has a safety profile no worse than that of complete submuscular coverage but offers the benefit of fewer expansions and the potential for more predictable secondary revisions.

How well did you know this?
1
Not at all
2
3
4
5
Perfectly
12
Q

Pros/cons of ADM and prosthetic reconstruction

A

ADM has enhanced prosthesis-based reconstruction and remains useful in immediate prosthetic breast reconstruction. It has been reported to decrease capsule contracture. It also allows for greater initial fill of saline, potentially improving cosmetic outcomes.

However, it has been found to have higher rates of postoperative seroma and infection.

How well did you know this?
1
Not at all
2
3
4
5
Perfectly
13
Q
A 42-year-old woman is evaluated because of an invasive cancer of the right breast. Which of the following best approximates the likelihood that this patient's cancer is associated with the BRCA1 or BRCA2 genes?
A) 1%
B) 10%
C) 25%
D) 40%
E) 75%
A

B) 10%

How well did you know this?
1
Not at all
2
3
4
5
Perfectly
14
Q

Among familial breast cancers, _____ are considered hereditary

A

Among familial breast cancers, 5 to 10% are considered to be hereditary.

How well did you know this?
1
Not at all
2
3
4
5
Perfectly
15
Q

Breast cancer susceptibility genes

A

The breast cancer susceptibility genes (BRCA) belong to a class of genes known as tumor suppressors. In normal cells, BRCA1 and BRCA2 genes stabilize the DNA and prevent uncontrolled cell growth.

A woman’s lifetime risk of developing breast and/or ovarian cancer is greatly increased if she inherits a mutation on BRCA1 or BRCA2 genes.

How well did you know this?
1
Not at all
2
3
4
5
Perfectly
16
Q

BRCA-1 and -2 breast cancers occur in what age group?

A

BRCA1-and BRCA2-related breast cancers are often associated with estrogen receptor-negative tumors

How well did you know this?
1
Not at all
2
3
4
5
Perfectly
17
Q

BRCA-1 and -2 breast cancers are associated with estrogen receptor - _________ tumors

A

BRCA1-and BRCA2-related breast cancers are often associated with estrogen receptor-negative tumors

How well did you know this?
1
Not at all
2
3
4
5
Perfectly
18
Q
A 37-year-old woman comes to the office for consultation regarding left breast reconstruction after mastectomy to treat breast cancer. Chemotherapy and radiation therapy are planned postoperatively. She wears a size 34D brassiere. Height is 5 ft 6 in (168 cm), and weight is 160 lb (73 kg). BMI is 25.8 kg/m2. She does not want abdominal scars. Autologous reconstruction with a transverse musculocutaneous gracilis (TMG) flap is planned. Which of the following is a disadvantage of using a TMG flap for this patient's reconstruction?
A) Difficulty of flap harvest
B) High risk of donor site morbidity
C) High risk of fat necrosis
D) Small flap size
E) Variable vascular anatomy
A

D) Small flap size

How well did you know this?
1
Not at all
2
3
4
5
Perfectly
19
Q

The size of a transverse musculocutaneous gracilis flap for breast reconstruction

A

The largest flap harvested in one large series weighed 420 g and measured 30 × 10 cm. The flap usually offers enough volume to reconstruct small-to mid-sized breasts.

How well did you know this?
1
Not at all
2
3
4
5
Perfectly
20
Q

The major advantages of a transverse musculocutaneous gracilis flap for breast reconstruction:

A

The major advantage of this flap as compared with other flaps, such as the gluteal flap or the perforator latissimus flap, is its constant vascular anatomy. Flap perfusion is always reliable, with low rates of fat necrosis and tissue similar in consistency with breast tissue. The donor scar is inconspicuous. The incision is comparable to incisions for a thigh lift and is well hidden. A distortion of the labia majorais typically not observed.

How well did you know this?
1
Not at all
2
3
4
5
Perfectly
21
Q

Functional morbidity after transfers myocutaneous gracilis flap harvest

A

functional donor-site morbidity after TMG flap harvest itself is low.

How well did you know this?
1
Not at all
2
3
4
5
Perfectly
22
Q
A 43-year-old Caucasian woman is referred to the office because of a mass on her right breast that has been rapidly growing for 8 weeks. Physical examination shows a 4.5-cm, freely movable mass in the right breast. No axillary adenopathy or nipple discharge is noted. Which of the following is the most likely nonepithelial neoplasm in this patient?
A) Fibroadenoma
B) Hamartoma
C) Lipoma
D) Phyllodes tumor
E) Primary breast lymphoma
A

D) Phyllodes tumor

How well did you know this?
1
Not at all
2
3
4
5
Perfectly
23
Q

Most common non epithelial neoplasm of the breast

A

Phyllodes tumor

How well did you know this?
1
Not at all
2
3
4
5
Perfectly
24
Q

Phyllodes tumor

A

Phyllodes tumors represent about 1% of tumors in the breast; they are the most commonly occurring nonepithelial neoplasm of the breast. The tumor has a smooth, sharply demarcated texture and is generally freely movable. It is a relatively large tumor, with an average size of 5 cm

How well did you know this?
1
Not at all
2
3
4
5
Perfectly
25
Q

Hamartoma of the breast

A

Hamartomas of the breast are benign tumors composed primarily of dense fibrous tissues with variable amounts of fat and associated ducts.

How well did you know this?
1
Not at all
2
3
4
5
Perfectly
26
Q

Percent of phyllodes tumor that are benign / malignant

A

Eighty-five percent of phyllodes tumors are benign, and 15% are malignant.

How well did you know this?
1
Not at all
2
3
4
5
Perfectly
27
Q

Who gets phyllodes tumors?

A

There is no race predilection; occur almost exclusively in women

How well did you know this?
1
Not at all
2
3
4
5
Perfectly
28
Q

In young women under age 25 years, asymmetric, tender, and fibrocystic tissues usually point to:

A

In young women under age 25 years, asymmetric, tender, and fibrocystic tissues usually point to a fibroadenoma or circumscribed fibrocystic mass.

How well did you know this?
1
Not at all
2
3
4
5
Perfectly
29
Q

A 12-year-old girl with Beckwith-Wiedemann syndrome develops profound breast enlargement at the onset of puberty. Physical examination shows two distinct masses in the right breast and severe hyperplasia consuming the left breast. A photograph is shown. Which of the following is the most appropriate surgical management?
A) Hormone suppression therapy
B) Right lumpectomies; left mastectomy with skin reduction with application of nipple as skin graft
C) Right simple mastectomy; left mastectomy with skin reduction with application of nipple as skin graft
D) Right simple mastectomy; left simple mastectomy with sentinel lymph node biopsy
E) Right total mastectomy; left modified radical mastectomy

A

B) Right lumpectomies; left mastectomy with skin reduction with application of nipple as skin graft

Epithelial hyperplasia is a benign pathological process. Therefore, modified radical mastectomy or sentinel lymph node biopsy would not be indicated in the absence of cancer. Mastectomy on the right is not indicated because the masses are discrete and separate from the normal breast parenchyma. As such, a viable breast mound can be preserved in this 12-year-old girl. Because ofthe expansive nature of her hyperplasia on the left, skin reduction is required to ensure smooth contour of her skin flaps. The nipple may be spared because of the benign disease. Reconstruction can be performed in a delayed fashion after development is complete.

How well did you know this?
1
Not at all
2
3
4
5
Perfectly
30
Q

Beckwith-Wiedemann syndrome

A

Congenital overgrowth disorder, w/ magroglossia, macrosomia, midline abdominal defects, ear creases or pits, and neonatal hypoglycemia; increased risk of childhood cancers, esp Wilms tumor, pancreatoblastoma, hepatoblastoma

How well did you know this?
1
Not at all
2
3
4
5
Perfectly
31
Q

A 60-year-old woman receives low-molecular-weight heparin (LMWH) 40 U subcutaneously 1 hour before undergoing breast reconstruction using a unilateral transverse rectus abdominis musculocutaneous (TRAM) flap. Weight is 185 lb (84 kg); BMI is 32 kg/m2. Which of the following is the most likely effect of the LMWH on perioperative risks in this patient?
A ) Decreased risk of flap failure
B ) Decreased risk of postoperative hematoma
C ) Decreased risk of pulmonary thromboembolism
D ) Increased risk of blood transfusion
E ) Increased risk of infection

A

C ) Decreased risk of pulmonary thromboembolism

How well did you know this?
1
Not at all
2
3
4
5
Perfectly
32
Q

The risk of developing DVT begins with:

A

As the risk of developing DVT begins with anesthesia induction, it is generally recommended that heparin therapy be started before surgery.
.. Out of concern for bleeding, some surgeons start heparin therapy in the early postoperative period.

How well did you know this?
1
Not at all
2
3
4
5
Perfectly
33
Q
A 53-year-old woman comes to the office because of ulcerated tissue 6 weeks after undergoing radiation therapy for breast cancer. She underwent mastectomy 1 year ago. Analysis of the radiated tissue is most likely to show an increase in which of the following?
A ) Acute inflammatory response
B ) Cytokines and growth factors
C ) Neutrophil function
D ) Tissue oxygenation
E ) Vessel thrombosis
A

E ) Vessel thrombosis

How well did you know this?
1
Not at all
2
3
4
5
Perfectly
34
Q

Clinical appearance of acute radiation treatment

A

In acute radiation injury, the skin becomes erythematous and edematous with the dilation of fine blood vessels, endothelial edema, and lymphatic obliteration.

How well did you know this?
1
Not at all
2
3
4
5
Perfectly
35
Q

Skin perfusion in acute radiation injury

A

Although perfusion of the skin assessed with fluorescein injection appears normal, tissue oxygenation is inadequate.

How well did you know this?
1
Not at all
2
3
4
5
Perfectly
36
Q

Cells/ healing with acute radiation injury

A

Healing is impaired with slowed fibroblast proliferation and impairment of the acute inflammatory response. Fibroblast defects are the main problem in the inhibited healing of chronic radiation injury. Phagocytosis and bacteriocidal metabolic functions in neutrophils are also impaired.

How well did you know this?
1
Not at all
2
3
4
5
Perfectly
37
Q

A 45-year-old woman who has breast cancer comes to the office for consultation regarding bilateral breast reconstruction. Reconstruction using autologous abdominal tissue is considered. The risk of abdominal morbidity is discussed. Which of the following flap techniques is most likely to result in the lowest level of overall abdominal morbidity?
A ) Deep inferior epigastric artery perforator
B ) Free muscle-sparing transverse rectus abdominis musculocutaneous (TRAM)
C ) Free TRAM
D ) Pedicled TRAM
E ) Superficial inferior epigastric artery flap

A

E ) Superficial inferior epigastric artery flap

How well did you know this?
1
Not at all
2
3
4
5
Perfectly
38
Q

Which abdominal flap results in the lowest overall abdominal morbidity

A

The superficial inferior epigastric artery (SIEA) flap results in the lowest level of overall abdominal morbidity, as the technique used in harvesting this flap leaves the abdominal fascia intact.

How well did you know this?
1
Not at all
2
3
4
5
Perfectly
39
Q

Who can receive a superficial inferior epigastric artery flap?

A

SIEA vessels are only present in less than one third of patients, and only one half of those patients will have vessels of sufficient diameter to support a free tissue transfer.

How well did you know this?
1
Not at all
2
3
4
5
Perfectly
40
Q

Cons of the SIEA flap

A

The SIEA flap is associated with a higher frequency of total flap loss, in addition to a higher incidence of fatty necrosis

How well did you know this?
1
Not at all
2
3
4
5
Perfectly
41
Q

How can the DIEP flap cause abdominal wall morbidity?

A

The deep inferior epigastric artery perforator (DIEAP or DIEP) flap involves dissection of one or two (occasionally more) perforators through the rectus muscle to the inferior epigastric vessels. Although this technique does not include any rectus muscle or sheet/fasica within the flap itself, it does involve moderate-level trauma to those organs and can cause abdominal wall morbidity.

How well did you know this?
1
Not at all
2
3
4
5
Perfectly
42
Q

Pedicled TRAM morbidity vs other flaps

A

The general consensus remains thatin a bilateral reconstruction, pedicled TRAM flaps are associated with higher levels of overall abdominal morbidity (hernias, bulges, weakness, intolerance to exercise, etc) when compared with the use of MS free TRAM, DIEP, or SIEA flaps.

How well did you know this?
1
Not at all
2
3
4
5
Perfectly
43
Q
A 45-year-old man comes to the office for consultation regarding breast cancer after undergoing gene testing and learning that he is a carrier of the BRCA2 gene mutation. He has a strong family history of breast, prostate, and ovarian cancers. Which of the following best represents his lifetime risk for developing breast cancer? 
A ) 6% 
B ) 15% 
C ) 30% 
D ) 60% 
E ) 85%
A

A ) 6%

How well did you know this?
1
Not at all
2
3
4
5
Perfectly
44
Q

The relationship between male breast cancer and a deleterious ______ mutation has been well established:

A

The relationship between male breast cancer and a deleterious BRCA2 mutation has been well established. In the largest study to date, the lifetime risk of developing breast cancer in a BRCA2 male carrier is approximately 7%.

How well did you know this?
1
Not at all
2
3
4
5
Perfectly
45
Q

The relative risk of developing breast cancer is highest for men in their _________, and it ________ with increasing age

A

The relative risk of developing breast cancer is highest for men in their 30s and 40s, and it decreases with increasing age

How well did you know this?
1
Not at all
2
3
4
5
Perfectly
46
Q

A 21-year-old woman comes to the office for consultation regarding a palpable lump in the left breast that she first noticed 4 months ago. Phyllodes tumor of the breast is diagnosed. Which of the following is the most appropriate management at this time?
A ) Observation and incisional biopsy
B ) Neoadjuvant chemotherapy
C ) Local excision and annual surveillance
D ) Wide local excision and radiation therapy
E ) Mastectomy with immediate reconstruction

A

C ) Local excision and annual surveillance

How well did you know this?
1
Not at all
2
3
4
5
Perfectly
47
Q

Need for follow up of phyllodes tumor:

A

!nnual surveillance and follow-up is recommended because the pathologic appearance does not always predict the clinical behavior.

How well did you know this?
1
Not at all
2
3
4
5
Perfectly
48
Q

A 29-year-old woman who is positive for the BRCA1gene underwent bilateral mastectomies and immediate reconstruction with placement of tissue expanders. After completion of the second session of tissue expansion, she has severe pain in the chest wall. There is no evidence of hematoma or infection. Which of the following is the most appropriate next step in management?
A ) Administer pain medication
B ) Apply a warm compress to the breasts
C ) Plan for return to the operating room
D ) Reassure the patient
E ) Withdraw fluid from the tissue expander

A

E ) Withdraw fluid from the tissue expander

The patient’s pain immediately after expansion (or even in a slightly delayed manner) is indicative of ischemic tissue with poor oxygen tension.

Neural tissue has been shown to have the lowest tolerance to ischemia, hence in part explaining the pain. The most appropriate step in management is to withdraw enough fluid to relieve pain. This amount can be dramatically small at times; a few milliliters can correct the problem.

How well did you know this?
1
Not at all
2
3
4
5
Perfectly
49
Q

_______ issue has been shown to have the lowest tolerance to ischemia.

A

Neural tissue has been shown to have the lowest tolerance to ischemia

How well did you know this?
1
Not at all
2
3
4
5
Perfectly
50
Q

A 48-year-old woman undergoes skin-sparing mastectomy followed by immediate breast reconstruction with implantation of subpectoral prostheses. Which of the following best describes the resulting effect on pectoralis torque strength in this patient?
A ) Decreased on the operated side
B ) Decreased on the side of hand dominance
C ) Increased on the operated side
D ) Increased on the side of hand dominance
E ) No effect

A

A ) Decreased on the operated side

How well did you know this?
1
Not at all
2
3
4
5
Perfectly
51
Q

Effect of immediate subpectoral prosthetic breast reconstruction following skin-sparing mastectomy on pectoralis strength

A

The effect of immediate subpectoral prosthetic breast reconstruction following skin-sparing mastectomy on pectoralis function is a decrease in torque strength on the operated side by approximately 20% compared to the nonoperated side. This loss represents approximately half of the torque needed by healthy subjects over 50 years of age to rise from a chair and was therefore considered substantial (in the study showing this).

How well did you know this?
1
Not at all
2
3
4
5
Perfectly
52
Q

An 18-year-old man with Klinefelter syndrome is referred to the office because of overdeveloped breasts. Diagnostic workup testing is ordered. The likelihood of breast cancer is increased if testing shows which of the following results?
A ) Decreasedlevel of plasma follicle-stimulating hormone
B ) Decreased level of plasma luteinizing hormone
C ) High number of progesterone receptors in the mesenchymal breast tissue
D ) Increased level of plasma testosterone
E ) Low number of estrogen receptors in the mesenchymal breast tissue

A

C ) High number of progesterone receptors in the mesenchymal breast tissue

How well did you know this?
1
Not at all
2
3
4
5
Perfectly
53
Q

Breast size in patients with Klinefelter syndrome

A

Approximately one half of patients with typical Klinefelter syndrome (47,XXY) and one third of those with the mosaic variety (46,XY/47,XXY) develop gynecomastia.

How well did you know this?
1
Not at all
2
3
4
5
Perfectly
54
Q

Patients with Klinefelter syndrome who develop gynecomastia have:

A

Patients with Klinefelter syndrome and exhibiting gynecomastia have elevated plasma follicle-stimulating and luteinizing hormone levels, along with decreased plasma testosterone levels, suggesting a hormonal link with gynecomastia in the Klinefelter population.

How well did you know this?
1
Not at all
2
3
4
5
Perfectly
55
Q

Potential mechanism by which Patients w/ Klinefelter syndrome may develop breast neoplasms

A

The presence of estrogen and progesterone receptors in elevated concentration in patients with Klinefelter syndrome provides a potential mechanism by which these patientsdevelop breast neoplasms.

The absence of elevated estrogen and progesterone receptors in patients with idiopathic gynecomastia might help to clarify why these patients rarely develop breast malignancies.

How well did you know this?
1
Not at all
2
3
4
5
Perfectly
56
Q

Cause of most cases of gynecomastia

A

idiopathic

How well did you know this?
1
Not at all
2
3
4
5
Perfectly
57
Q

Hormones associated with development of gynecomastia

A

Studies have shown strong evidence for the estrogen-stimulating effects of breast tissue development and support for an inhibitory androgenic effect. Decreases in the androgen-to-estrogen ratio have also been associated with development of gynecomastia. No clear etiologic classification has been suggested based solely on hormonal influences.

How well did you know this?
1
Not at all
2
3
4
5
Perfectly
58
Q
A 36-year-old woman with a confirmed diagnosis of ductal carcinoma in situ is considering skin-sparing mastectomy and immediate reconstruction. The patient should be informed that her risk of developing recurrent ipsilateral breast cancer over the next four years is approximately which of the following?
A ) 2%
B ) 5%
C ) 10%
D ) 15%
E ) 20%
A

A ) 2%

How well did you know this?
1
Not at all
2
3
4
5
Perfectly
59
Q

Recurrence rate of ductal carcinoma after skin sparing mastectomy over next four years

A

Several studies support skin-sparing mastectomy as an oncologically safe technique, based on the absence of breast ductal epithelium at the margin of the native skin flaps. A recurrence rate of less than 2% at 45 monthsis quoted in a study by Slavin and colleagues

How well did you know this?
1
Not at all
2
3
4
5
Perfectly
60
Q
A 32-year-old woman with multifocal ductal carcinoma in situ in her right breast is scheduled to undergo mastectomy. There is no family history of breast cancer. Results of testing for the BRCA gene mutation are negative. MRI of the left breast shows no abnormalities. A discussion is held with the patient about prophylactic mastectomy on the contralateral breast. Which of the following is the principal factor in the determination of whether to perform bilateral mastectomy?
A ) Medical clearance
B ) Need for radiation
C ) Patient wishes
D ) Reconstructive options
E ) Tumor burden
A

C ) Patient wishes

In several studies, patients who underwent prophylactic mastectomy at the suggestion of the physician were generally more regretful than those who underwent the procedure following a patient-initiated discussion. The lack of perioperative emotional support also affected these patients. Lack of nipple sensation and inability to breast-feed are two factors leadingto patients’ regret about their decision

How well did you know this?
1
Not at all
2
3
4
5
Perfectly
61
Q
A 39-year-old woman is referred for consultation regarding reconstruction of the left breast two years after mastectomy for breast cancer. Implantation of a silicone prosthesis and reconstruction with a pedicled latissimus dorsi musculocutaneous flap are planned. This patient will most likely show deficits in which of the following shoulder functions?
(A)Extension and abduction
(B)Extension and adduction
(C)Flexion and abduction
(D)Flexion and adduction
(E)No deficits
A

(B)Extension and adduction

How well did you know this?
1
Not at all
2
3
4
5
Perfectly
62
Q

Functional deficits after latissimus doors musculocutaneous flap

A

Transfer of the latissimus dorsi musculocutaneous flap is associated withdeficits in extension and adduction.

How well did you know this?
1
Not at all
2
3
4
5
Perfectly
63
Q

Primary contribution (motion) of the latissimus doors muscle

A

The primary contribution of the muscle is in shoulder extension, adduction, and medial rotation.

64
Q

The __________ compensates for loss of the latissimus muscle

A

The synergistic action of the teres major muscle compensates for loss of the latissimus muscle. This leads to teres major hypertrophy in the long term.

65
Q
When performing a delayed autologous breast reconstruction, which of the following is the most appropriate placement of the skin paddle on the chest wall for optimal aesthetic appearance?
(A)Inferior and lateral
(B)Inferior and medial
(C)Superior and lateral
(D)Superior and medial
A

(A)Inferior and lateral

This placement creates a natural ptotic breast shape. Placement of the skin paddle in the medial or superior position results in an unaesthetic pattern with scars that may be difficult to hide in clothing. Placement of the skin paddle centrally, within the old mastectomy scar, also results in a less aesthetic contour.

66
Q

When considering aesthetic subunits of the breast during immediate breast reconstruction, the ideal skin paddle placement:

A

When considering aesthetic subunits of the breast during reconstruction, the ideal skin paddle placement and shape is areolar or expanded areolar for an immediate reconstruction.

67
Q

When considering aesthetic subunits of the breast during delayed breast reconstruction, the ideal skin paddle placement:

A

For delayed reconstruction, the ideal aesthetic subunits are lower outer crescent, lower half of the breast, inferolateral crescent, or entire breast.

68
Q
A 40-year-old woman comes to the office for consultation regarding breast reconstruction two years after undergoing modified radical mastectomy for breast cancer. Physical examination shows atrophy of the lower lateral pectoralis major muscle. Transection of which of thefollowing nerves during mastectomy is the most likely cause of the atrophy?
(A)Intercostobrachial
(B)Lateral intercostal
(C)Long thoracic
(D)Medial pectoral
(E)Thoracodorsal
A

(D)Medial pectoral

69
Q

The pectorals major is innervated by:

A

The pectoralis major muscle is innervated by branches from the brachial plexus. The lateral pectoral nerve, and the medial pectoral nerve

The medial and lateral pectoral nerves are named for the brachial plexus cord of their origin, not for the portion of the muscle that they supply

70
Q

The medial and lateral pectoral nerves are named for:

A

The medial and lateral pectoral nerves are named for the brachial plexus cord of their origin, not for the portion of the muscle that they supply

71
Q

Course of the lateral pectoral nerve

A

The pectoralis major muscle is innervated by branches from the brachial plexus (C5-6). The lateral cord gives off the lateral pectoral nerve, which crosses the axillary artery and vein and pierces the deep surface of the muscle. It supplies the medial and superior portions of the muscle.

72
Q

Course of the medial pectoral nerve

A

The pectoralis major muscle is innervated by branches from the brachial plexus (C8-T1). The medial cord gives off the medial pectoral nerve, which enters the deep surface of the pectoralis minor and then passes around the edge of the lateral pectoralis major muscle. At this point, this nerve is vulnerable to division during mastectomy.

73
Q

The second intercostobrachial nerve supplies :

A

The second intercostobrachial nerve supplies the skin of the axilla and inner arm. It is often transected or transiently injured during axillary dissection

74
Q

The lateral intercostal nerves supply:

A

The lateral intercostal nerves supply the breast

75
Q

The fourth intercostal nerve supplies:

A

The fourth intercostal nerve supplies nipple and areola sensation.

76
Q

The long thoracic nerve comes off the _____________ and supplies:

A

The long thoracic nerve comes off the lateral and posterior cords and supplies the serratus anterior. Division of this nerve results in a winged scapula.

77
Q

The thoracodorsal nerve comes from the ____________ and supplies:

A

The thoracodorsal nerve comes from the posterior cord and supplies the latissimus dorsi muscle

78
Q
A 43-year-old woman is scheduled for reconstruction of the right breast with a latissimus dorsi flap. She is at greatest risk for which of the following complications?
(A)Chronic chest wall pain
(B)Dorsal wound dehiscence
(C)Partial flap necrosis
(D)Seroma
(E)Stiffness in theipsilateral shoulder
A

(D)Seroma

79
Q

Frequency of serum formation after latissimus dorsi breast reconstruction

A

The most common complication is seroma formation, which is estimated to be as high as 35% to 60%.

80
Q

Most common complication after latissimus dorsi breast reconstruction

A

The most common complication is seroma formation, which is estimated to be as high as 35% to 60%.

81
Q

Common complications after latissimus breast reconstruction

A

The most common complication is seroma formation, which is estimated to be as high as 35% to 60%. ther less common complications include chest wall pain (6%), dorsal wound dehiscence (4%), partial flap necrosis (1%–7%), and shoulder stiffness (1%).

82
Q

A 58-year-old nulligravid woman who is scheduled to undergo bilateral prophylactic mastectomy comes to the office for consultation regarding immediate breast reconstruction. She works full time as a fitness instructor. The patient currently wears a size 34B brassiere and wants her brassiere size to be increased to a C cup, but she wants to make sure that scarring is minimized. Height is 5 ft 5 in and weight is 120 lb. Physical examination of the abdomen shows paucity of extra tissue. Which of the following is the most appropriate breast reconstruction procedure for this patient.
(A)Bilateral autogenous reconstruction
(B)Bilateral tissue expansion followed by implantation of prostheses
(C)Delayed breast reconstruction after pathology is confirmed
(D)Single-stage reconstruction with prostheses
(E)TRAM flap followed by implantation of prostheses

A

(B)Bilateral tissue expansion followed by implantation of prostheses

Bilateral latissimus dorsi or TRAM flaps may not be the first choice for a fitness instructor who will most likely require prostheses. The recovery time and the associated donor site morbidity may prevent her from returning to the gym as an instructor. Tissue expander placement and placement of prostheses can often be done through the same incisions used for extirpation and as such havethe least amount of visible scar. No additional tissue is used to create the breast mound. Immediate breast reconstruction can offer some psychological benefits for the woman and may be associated with fewer surgeries overall. Prostheses placed without anexpanded pocket may not be as reliable in reconstruction of moderate-sized breasts

83
Q

A 16-year-old girl is brought to the office by her parents because she has had worsening pain in the right breast (shown) for the past eight months. Ten years ago, she sustained a full-thickness burn to the back, right chest, and abdomen requiring excision and split-thickness skin grafting. Physical examination shows contracture of the scar and lateral expansion of the breast. After release of the burn scar contractures, a defect of 200 cm2is created. Which of the following is most appropriate for coverage of this defect?
(A)Dermal regeneration template (Integra)
(B)Latissimus dorsi musculocutaneous flap
(C)Meshed split-thickness skin graft
(D)Nonmeshed thin split-thickness skin graft
(E)Z-plasty release of the constricted scar

A

(A)Dermal regeneration template (Integra)

The patient described is suffering from pain caused by a developing breast that is constricted under a nonexpanding split-thickness skin graft placed at the time of the burn excision. The burn scar contractures are first released to allow expansion of the underlying breast. The decision for coverage material is then based on what will most likely permit further expansion of the breast.Dermal regeneration template (Integra) has been shown to allow secondary expansion and is the most appropriate choice for the scenario described. The wound is covered with an expandable dermal layer, which is then covered again by a split-thickness skin graft, leaving a minimal donor scar.

84
Q

Integra and expansion

A

Dermal regeneration template (Integra) has been shown to allow secondary expansion. A wound is covered with an expandable dermal layer, which is then covered again by a split-thickness skin graft, leaving a minimal donor scar.

85
Q

A 55-year-old woman undergoes modified radical mastectomy with immediate first-stage reconstruction of the right breast with a tissue expander. Before the second stage to exchange the tissue expander with a permanent prosthesis is initiated, pathology results from analysis of tissue from the right breast indicate metastatic carcinoma of four axillary lymph nodes. Radiation therapy is recommended. Which of the following interventions will result in the best long-term appearance of the reconstructed breast?
(A)Complete the tissue expansion before radiation and exchange the tissue expander with a prosthesis after radiation
(B)Deflate the tissue expander before radiation and reinflate the tissue expander and exchange with a prosthesis after radiation
(C)Remove the tissue expander and reconstruct the breast with a TRAM flap before radiation
(D)Remove the tissue expander before radiation; after radiation, reinsert and expand a tissue expander and then exchange with a prosthesis
(E)Remove the tissue expander before radiation and reconstruct the breast with a TRAM flap after radiation

A

(E)Remove the tissue expander before radiation and reconstruct the breast with a TRAM flap after radiation

86
Q

Breast reconstruction after radiation therapy is best accomplished via:

A

Breast reconstruction after radiation therapy is best accomplished with autologous tissue such as a TRAM flap.

ny reconstruction method that uses prostheses or tissue expanders after radiation therapy has a higher risk of complications and poor cosmetic outcome when compared with reconstruction using autologous tissue.

87
Q

Radiation and prostheses

A

Radiation increases the risk of infection and exposure of the prosthesis, leading to failure of breast reconstruction procedures using prostheses. Radiation also increases the risk for capsular contracture, resulting in a poor aesthetic result.

88
Q

A 45-year-old woman with T3 N0 invasive ductal carcinoma in the inferior pole of the left breast is scheduled to undergo segmental mastectomy and subsequent radiation therapy. She currently wears a size 36DDD brassiere and is willing to accept any cup size from C to DDD. Which of the following interventions will yield the best cosmetic result in this patient?
(A)Bilateral reduction mammaplasty
(B)Implantation of a prosthesis in the left breast and mastopexy of the right breast
(C)Latissimus dorsi musculocutaneous flap reconstruction of the left breast and mastopexy of the right breast
(D)TRAM reconstruction of the defect
(E)No reconstruction

A

(A)Bilateral reduction mammaplasty

In a patient with large breasts, in whom a partial mastectomy is required, reduction mammaplasty is a very good option. This procedure will potentially relieve symptoms of macromastia, reduce the amount of breast tissue present in both breasts, and offer the best aesthetic outcome

89
Q
A 40-year-old woman is scheduled to undergo reconstruction of the right breast via a free TRAM flap. She has smoked two packs of cigarettes daily for the pasteight years. This patient=s smoking history increases her risk of which of the following postoperative complications?
(A)Hematoma
(B)Mastectomy flap necrosis
(C)Seroma
(D)TRAM flap loss
(E)Vessel thrombosis
A

(B)Mastectomy flap necrosis

90
Q

A 48-year-old woman comes to the office for consultation regarding reconstruction ofthe right breast after mastectomy because of cancer. The patient is concerned about maximizing the aesthetic result and minimizing any donor site deformity. Physical examination shows a well-healed chest wall and a B-cup left breast with Grade 3 ptosis. Soft-tissue reconstruction with a superior gluteal artery perforator (SGAP) free flap is planned. Which of the following is a disadvantage of this procedure?
(A)Difficulty molding the gluteal fat
(B)Gait dysfunction
(C)Inability to fully hide the donor scar
(D)Inability to provide a sensate flap
(E)Lack of abundant soft tissue

A

(A)Difficulty molding the gluteal fat

91
Q

Issue with a superior gluteal artery perforator flap for breast reconstruction

A

In soft-tissue reconstruction with a superior gluteal artery perforator (SGAP) free flap, the firmer consistency of gluteal fat causes difficulty in molding tissue, particularly in breast reconstruction. Secondary procedures may be necessary to obtain the desired shape.

92
Q

Gait dysfunction after superior gluteal artery perforator flap

A

Gait dysfunction is extremely uncommon after a SGAP flap if care is taken to avoid injuryto the motor branch.

93
Q

Scar after superior gluteal artery perforator flap

A

The scar is generally well hidden by clothing and the donor site deformities are generally minimal. Even in thin patients there is an abundance of adipose tissue in the gluteal area.

94
Q

Sensation to a superior gluteal artery perforator flap

A

Although not commonly done, a sensate branch of thenervi clunium superiores can provide sensation to the flap.

95
Q
A healthy 27-year-old woman comes to the office for follow-up examination three months after undergoing bilateral prophylactic mastectomy and reconstruction with saline prostheses. She is concerned about the incision on her left breast because it is slightly swollen and warm to the touch. Physical examination shows mild erythema of the left breast and normal healing of the right breast. Which of the following is the most appropriate initial management?
(A) Observation
(B) Oral antibiotic therapy
(C) Open lavage of the implant pocket
(D) Removal of the prosthesis
(E) Exchange of the prosthesis
A

(B) Oral antibiotic therapy

In a situation such as this, in which a patient has a minor infection and no actual or threatened exposure of the prosthesis, antibiotics alone are the treatment of choice. Follow-up certainly would be necessary because a mild infection could become severe if not properly treated.

96
Q
In reconstruction mammaplasty after mastectomy, which of the following is the most significant advantage of self-filling osmotic tissue expanders over conventional tissue expanders?
(A) Anatomic shape
(B) Incidence of infection
(C) Permanent implantation
(D) Short filling phase 
(E) Subcutaneous placement
A

(B) Incidence of infection

97
Q

Infection w/ osmotic tissue expanders

A

Because there is no need for external fillings, the risk of infection and painful injections is obviated with self-filling osmotic tissue expanders, which contain osmotic-active hydrogel vinyl pyrrolidine and methylmethacrylate (developed by Wiese and produced by Osmed in Elman, Germany).

98
Q

Improvements in 2nd generation of osmotic expanders

A

The second-generation expander has a silicone membrane with small pores enclosing the osmotic hydrogel. This membrane reduces expansion speed, and final volume is accurately defined. It has been successfully used in delayed reconstruction mammaplasty, correction of breast anomalies (i.e., tubular breast, micromastia, etc.), and coverage of defects after excision of tumors, scars, burns, and alopecia

99
Q

General overview of osmotic tissue expanders / similarities to conventional expanders

A

As with conventional tissue expanders, self-filling osmotic expanders are not meant to be permanent, require submuscular placement, have a filling phase of 40 to 60 days, and require replacement with a permanent prosthesis in four to six months. The anatomically shaped osmotic expander is prone to rotation, resulting in an inferior result.

100
Q

A 45-year-old woman who underwent reconstruction of the left breast with a TRAM flap followed by radiation two years ago is evaluated for a second reconstruction procedure on the left breast (shown). She requests that only reconstruction with her own tissues be performed and is opposed to surgery of the right breast to restore symmetry. On physical examination, the breast is shrunken and firm. After removal of the unsalvageable left breast, which of the following is the most appropriate reconstruction technique at this time? (contralateral breast is very large)
(A) Deep inferior epigastric perforator (DIEP) flap
(B) Free TRAM flap
(C) Latissimus dorsi myocutaneous flap
(D) Superior gluteal artery perforator (SGAP) flap
(E) Thoracoepigastric flap

A

(D) Superior gluteal artery perforator (SGAP) flap

When there is insufficient lower abdominal tissue for tissue expansion and transfer of a second flap, neither a free TRAM flap nor a deep inferior epigastric perforator (DIEP) flap recruiting soft tissue inthe same lower abdominal areas is an option. A latissimus dorsi myocutaneous flap is a reliable source of autogenous tissue; however, considering the size of the contralateral breast in this patient, a latissimus dorsi myocutaneous flap would need to be supplemented by a permanent expander implant to provide sufficient volume.

A gluteus free flap is considered when the latissimus dorsi flap is not sufficient or available and the patient requests autogenous tissue breast reconstruction.

101
Q

Free sensate flap superior gluteal artery perforator flap

A

The free sensate superior gluteal artery perforator (SGAP) flap is a free sensate flap based on the superior gluteal artery perforator. It has an abundance of adipose tissue (specimens up to 760 g), even in thin patients, a relatively long vascular pedicle, a discrete scar, improved projection compared with the DIEP and TRAM flaps, and preservation of the entire gluteus maximusmuscle with extremely low donor site morbidity.

102
Q

For reconstruction mammaplasty, which of the following is an advantage of the extended latissimus dorsi flap over the standard latissimus dorsi myocutaneous flap?
(A) Better flap perfusion
(B) Decreased need for breast implant
(C) Fewer donor-site seroma
(D) Less sacrifice of latissimus dorsi muscle
(E) Smaller donor-site scar

A

(B) Decreased need for breast implant

103
Q

Extended latissimus dorsi flap

A

The design of the ELD flap has evolved to include the parascapular and scapular ―fat fascia‖ in addition to the lumbar fat for additional volume. The main advantage of the ELD flap is that it can provide autogenous tissue to replace breast volume without an implant and with acceptable donor site contour and scar. Because the ELD flap transports additional tissue from the back so a breast prosthesis is not required, this avoids all potential problems associated with implants, e.g., development of capsular contracture and deformation of the reconstructed breast

With ELD flaps, donor site complications, including seroma formation and wound necrosis, can be a significant problem.

104
Q

A 36-year-old woman comes to the office for consultation regarding breast reconstruction one year after she underwent right modified radical mastectomy. The procedure was followed by six weeks of radiation therapy. She has no history of other surgical procedures or serious medical illnesses. Height is 5 ft 4 in tall and weight is 135 lb. She wears a size 32B brassiere. Which of the following is the LEAST appropriate breast reconstruction procedure for this patient?
(A) Extended latissimus dorsi flap
(B) Latissimus dorsi flapwith saline-filled prosthesis
(C) Superior gluteal artery perforator (SGAP) flap
(D) TRAM flap
(E) Two-staged reconstruction with a tissue expander and saline-filled prosthesis

A

(E) Two-staged reconstruction with a tissue expander and saline-filled prosthesis

Radiation therapy has significant negative effects on the outcome of breast reconstruction with prostheses. There is a significant increase in capsular contracture and other complications, which are unrelated to implant type

105
Q

A 60-year-old woman with breast cancer is scheduled to undergo modified radical mastectomy followed by postoperative radiation therapy. Which of the following techniques will yield the most natural appearance of the breast with the fewest complications?
(A) Delayed reconstruction with an autologous tissue flap
(B) Delayed reconstruction with a tissue expander followed by implantation of a prosthesis
(C) Immediate reconstruction with an autologous tissue flap
(D) Immediate reconstruction with a tissue expander followed by implantation of a prosthesis

A

(A) Delayed reconstruction with an autologous tissue flap

106
Q

A 56-year-old woman comes to the office for routine follow-up 10 days after undergoing skin-sparing right modified radical mastectomy and immediate breast reconstruction with a subpectoral tissue expander. Physical examination shows a 2 x 3-cm area of frank necrosis of the lower lateral mastectomy skin flap. The tissue expander in this area is not covered by muscle, but it is not exposed and there are no signs of infection. Which of the following is the most appropriate next step in management?
(A) Admission to the hospital for intravenous administration of antibiotics and observation
(B) Debridement of necrotic tissue and primary closure
(C) Debridement of necrotic tissue and removal of the tissue expander
(D) Initiation of tissue expansion by instillation of 100 mL of saline into the tissue expander
(E) Oral administration of an antibiotic and follow-up in one week

A

(B) Debridement of necrotic tissue and primary closure

If the pectoralis major or serratus anterior muscle were covering the tissue expander in the necrotic area of the skin flap, this necrotic tissue could be managed nonoperatively by allowing it to slough and by performing regular dressing changes while secondary healing occurs.

For this patient, the most appropriate next step is to debride the necrotic tissue of the mastectomy skin flap using sterile technique and then perform primary closure of the resulting wound. This management minimizes the risk of infection and of exposure of the tissue expander.Oral or intravenous administration of an antibiotic does not prevent eventual exposure and infection of a breast tissue expander that is covered with nonviable soft tissue.

Removal of the tissue expander is not necessary because it is not infected at this time. If the skin flap necrosis can be debrided while the sterility of the tissue expander is maintained, the resulting wound can undergo primary closure and the breast reconstruction can be preserved.

107
Q
A 14-year-old girl has absence of the nipple and lack of development of the right breast. The left breast has normal shape and normal nipple-areola complex and fits a B-cup brassiere. Family history includes normal breast development in the parents and siblings. On physical examination, both pectoralis muscles are present and fully developed. No abnormalities of the hands are noted. Which of the following is the most likely diagnosis?
(A) Anterior thoracic hypoplasia
(B) Congenital absence of the breast 
(C) Hypoplasia of the breast
(D) Poland syndrome
(E) Tubular breast deformity
A

(B) Congenital absence of the breast

108
Q

Congenital absence of the breast

A

Congenital absence of the breast is defined by the absence of the nipple and mammary gland. This rare genetic condition is highly heterogeneous in presentation and inheritance.

109
Q

Anterior thoracic hypoplasia

A

Anterior thoracic hypoplasia is defined by unilateral sinking of the anterior chest wall, hypoplasia of the breast, superior location of the nipple-areola complex, and normal pectoralis muscles.

110
Q

Poland syndrome

A

This patient has Poland syndrome, which is characterized by unilateral breast or nipple hypoplasia, unilateral absence of the sternal head of the pectoralis major muscle, absence of the pectoralis minor muscle, and ipsilateral syndactyly or hypoplasia of the ipsilateral extremity. In severe Poland syndrome, rib anomalies also occur

111
Q

Tubular breast deformity

A

Tubular breast deformity involves constriction at the base of the breast, hypoplasia of the breast, and herniation of tissue into the nipple-areola complex.

112
Q
A 13-year-old girl is brought to the office by her parentsbecause her left breast is not developing. On examination, both nipples and areolae are present. The breasts are asymmetric; the left breast is considerably smaller. The left anterior axillary fold is absent. This patient is most likely to have which of the following additional developmental differences?
(A) Ambiguous genitalia
(B) Craniosynostosis
(C) Microtia
(D) Pectus excavatum
(E) Syndactyly
A

(E) Syndactyly

113
Q

A 56-year-old woman with recurrent cancer of the right breast and no evidence of distant metastatic disease is scheduled to undergo completion mastectomy five years after undergoing lumpectomy (segmental mastectomy) for stage II carcinoma with radiation. In this patient, risk of complications is highest with which of the following methods of immediate breast reconstruction?
(A) Free superior gluteal artery perforator (SGAP) flap
(B) Free TRAM flap
(C) Latissimus dorsi myocutaneous flap with implant
(D) Pedicled TRAM flap
(E) Tissue expansion

A

(E) Tissue expansion

114
Q
Two years after undergoing modified radical mastectomy on the right, a 36-year-old woman desires reconstruction with an implant. Physical examination shows minimal bulk in the inferior and lateral portions of the right pectoralis major muscle. The most likely cause is denervation of which of the following nerves during mastectomy?
(A) Fourth intercostal
(B) Lateral pectoral
(C) Long thoracic
(D) Medial pectoral
(E) Thoracodorsal
A

(D) Medial pectoral

The minimal bulk in the inferior and lateral portions of the right pectoralis major muscle in this patient most likely results from denervation of the medial pectoral nerve, which is injured often during mastectomy.

115
Q

Assessing the status of the pectorals muscles before augmentation

A

It is important for the plastic surgeon to assess the status of the pectoralis muscle before attempting implant augmentation; this can be accomplished by having the patient place her hands on her hips and contract her chest muscles

116
Q
In a 30-year-old woman who carries the BRCA-2 genetic mutation, the lifetime risk of development of breast carcinoma is closest to what percentage?
(A) 10%
(B) 20%
(C) 40%
(D) 60%
(E) 100%
A

(D) 60%

117
Q

In persons who carry the BRCA-1 or BRCA-2 genetic mutation, the lifetime risk for development of breast carcinoma varies from:

A

In persons who carry the BRCA-1 or BRCA-2 genetic mutation, the lifetime risk for development of breast carcinoma varies from 50% to 85%.

118
Q

The BRCA-2 locus is located on ______________

A

The BRCA-2 locus is located on chromosome 13.

119
Q

The BRCA-1 locus is located on ______________

A

The BRCA-1 locus is located on chromosome 17.

120
Q

Cancers liked to BRCA-1

A

BRCA-1, whose locus lies on chromosome 17, has also been linked to carcinoma of the ovaries, prostate, and colon.

121
Q

Cancers liked to BRCA-2

A

This mutation is associated with an increased risk for carcinoma of the breast, but not with ovarian carcinoma.

122
Q
Six hours after undergoing breast reconstruction with a free TRAM flap, a 35-year-old woman has swelling of the reconstructed area and bluish discoloration of the skin. A photograph is shown above. Capillary refill time is increased. Findings on Doppler ultrasonography of the flap are within normal limits.Which of the following is the most appropriate next step in management?
(A) Fluid resuscitation
(B) Anticoagulant therapy
(C) Release of sutures and observation
(D) Operative exploration
A

(D) Operative exploration

This 35-year-old woman who underwent breast reconstruction now exhibits signs of venous outflow obstruction, including increased capillary refill time, dark or bluish discoloration of the skin, and swelling of the affected area. In addition, complete venous obstruction does not always manifest as abnormal findings on Doppler ultrasonography, and the Doppler signal may indeed fall within normal limits in many affected patients. Therefore, in the presence of these clinical signs, the only appropriate option is to perform repeat operative exploration

123
Q
In a 48-year-old woman who recently underwent bilateral reduction mammaplasty, histologic evaluation of resected tissue shows findings consistent with invasive ductal carcinoma. Which of the following factors will best determine the most appropriate next step in the management of this patient? 
(A) Age of the patient
(B) Initial tumor margins
(C) Location of the tumor
(D) Total volume of tissue resected
(E) Tumor size
A

(B) Initial tumor margins

f the tumor was not completely excised during the original reduction mammaplasty procedure, completion mastectomy is recommended

124
Q

Occult breast carcinoma has been identified in as many as __________ of women undergoing reduction mammaplasty.

A

Occult breast carcinoma has been identified in as many as 2% of women undergoingreduction mammaplasty.

125
Q

Occult carcinoma has been found in _________ of women who underwent a “balancing reduction” following mastectomy. There is some discrepancy in the incidence of occult carcinoma related to the inclusion or exclusion of in situ lesions.

A

Occult carcinoma has been found in 4.6% of women who underwent a “balancing reduction” following mastectomy. There is some discrepancy in the incidence of occult carcinoma related to the inclusion or exclusion of in situ lesions.

126
Q

Women who have occult malignancies identified during breast reduction are more likely to be:

A

Women who have occult malignancies identified during breast reduction are more likely to be younger and to have lobular tumors without palpable lymph nodes.

127
Q
A 45-year-oldwoman is scheduled to undergo mastectomy of the right breast followed by reconstruction using a free TRAM flap. She has a 15 pack/year history of cigarette smoking. This patient is at increased risk for development of each of the following postoperative complications EXCEPT
(A) abdominal flap necrosis
(B) fat necrosis
(C) hernia
(D) mastectomy skin flap necrosis
A

(B) fat necrosis

128
Q

Smokers vs nonsmokers: no significant differences in complications for free TRAM

A

Studies have shown no significant increase in the rate of fat necrosis, flap loss, or vessel thrombosis in patients who smoked whencompared with nonsmokers

129
Q

A 36-year-old woman desires breast reconstruction one year after undergoing right modified radical mastectomy followed by radiation therapy. She is 5 ft 4 in tall, weighs 135 lb, and is otherwise healthy. The left cup size of her bra is 32B.Which of the following is the most appropriate reconstructive option in this patient?
(A) Reconstruction with a latissimus dorsi flap and a saline-filled implant
(B) Reconstruction with a TRAM flap
(C) One-stage reconstruction with a silicone gel-filled implant
(D) Two-stage reconstruction with a tissue expander and a saline-filled implant
(E) Two-stage reconstruction with a tissue expander and a silicone gel-filled implan

A

(B) Reconstruction with a TRAM flap

130
Q
A 55-year-old woman is scheduled to undergo right mastectomy for management of breast carcinoma. She has a 30 pack/year history of cigarette smoking. She would like to undergo breast reconstruction using a free TRAM flap at the time of the mastectomy procedure. This patient would be at significantly increased risk for development of which of the following complications?
(A) Anastomotic thrombosis
(B) Fat necrosis
(C) Mastectomy skin flap necrosis
(D) Partial TRAM flap loss
(E) Wound infection
A

(C) Mastectomy skin flap necrosis

131
Q
A 55-year-old woman is scheduled to undergo bipedicled TRAM flap reconstruction of the right breast. Vascular circulation to the umbilicus in this patient will depend primarily on which of the following structures?
(A) Deep inferior epigastric arteries
(B) Lateral umbilical ligament
(C) Ligamentum teres
(D) Subcutaneous fat
(E) Subdermal plexus
A

(C) Ligamentum teres

132
Q

In addition to some contributions from other sources, the umbilicus receives arterial inflow via these deep sources:

A

In addition to some contributions from other sources, the umbilicus receives arterial inflow by means of three distinct deep sources: perforating vessels from the deep inferior epigastric arteries, small vessels of the ligamentum teres hepaticum, and the medial umbilical ligament.

The skin of the umbilicus also receives flow from the subdermal plexus.

133
Q

During unilateral TRAM flap elevation, umbilical blood flow:

A

During unilateral TRAM flap elevation, umbilical blood flow generally remains robust because of a dominance of direct vessels from the contralateral deep epigastric circulation.

The skin of the umbilicus also receives flow from the subdermal plexus. However, this source of blood to the umbilicus is completely disrupted when the umbilicus is surgically islanded during TRAM flap elevation

134
Q

During bilateral TRAM flap elevation, the circulation to the umbilicus:

A

During bilateral TRAM flap elevation, the circulation to the umbilicus must depend on the small vessels of the ligamentum teres and medial umbilical ligament because the deep inferior epigastric artery is ligated bilaterally.

The skin of the umbilicus also receives flow from the subdermal plexus. However, this source of blood to the umbilicus is completely disrupted when the umbilicus is surgically islanded during TRAM flap elevation

135
Q
A 42-year-old woman comes to the office for treatment after receiving a diagnosis of cancer of the right breast. She has decided to undergo mastectomy of the right breast. Which of the following is a relative CONTRAINDICATION to nipple-sparing mastectomy?
A) Comedo-type breast tumor
B) Invasive lobular carcinoma
C) Subareolar tumor
D) Tumor location 3 cm from the nipple
E) Tumor size of 2.5 cm
A

C) Subareolar tumor

A relative contraindication to nipple-sparing mastectomy is a centrally located tumor. Although various authors have employed different distance criteria, it is generally accepted that patients whose tumors are within 2 cm of the nipple are not candidates for nipple-sparing mastectomy.

Nipple-sparing mastectomy is an appropriate option for high-risk patients undergoing prophylactic mastectomy and for patients diagnosed with breast cancer who meet certain criteria. Those criteria are: tumor size of 3 cm or less, at least 2 cm from the nipple, not multicentric, and with clinically negative nodes.

Comedo carcinoma of the breast is a type of ductal carcinoma in situ. It is considered to be an early stage of breast cancer, is confined to the ducts, and usually does not spread beyond. It is not a contraindication to nipple-sparing mastectomy.

Invasive lobular carcinoma originates from the breast lobules, may form a thickening of the breast tissue rather than a discrete mass, and is often bilateral. As long as it meets the above criteria, it is not a contraindication to nipple-sparing mastectomy.

Inflammatory breast cancer, Paget disease, and tumors infiltrating the skin are also not candidates for skin-sparing or nipple-sparing mastectomy, according to several authors.

In more recent studies, a tumor size of 3 cm or less appears to result in no increase in local or regional recurrence in nipple-sparing mastectomy compared with alternative surgical approaches. A tumor of 2.5 cm is not a contraindication to nipple-sparing mastectomy.

136
Q
Which of the following characteristics is correlated with increased risk of nipple-areola complex necrosis in nipple-sparing mastectomies with immediate reconstruction?
A) Autologous tissue reconstruction
B) Direct to implant reconstruction
C) Patient age
D) Periareolar incision
E) Small breast size
A

D) Periareolar incision

Nipple-sparing mastectomies (NSMs) are becoming more common for both therapeutic and prophylactic mastectomies. Nipple-areola complex (NAC) necrosis can imperil reconstructive efforts, as well as negatively affect patients emotionally. It is important to maximize perfusion to the mastectomy skin flaps and NAC while still performing an oncologically sound procedure. There are multiple different incisions for performing NSM. Periareolar, inframammary-fold, radial, and vertical incisions are the most common. Periareolar incisions are associated with an increased risk of NAC necrosis in NSMs. Type of reconstruction, small breast size, and patient age have not been shown to be linked to increased rates of NAC necrosis.

137
Q

A 52-year-old woman receives a diagnosis of invasive ductal carcinoma of the right breast. Which of the following details from this patient’s history is the strongest risk factor for this diagnosis?
A) Early first pregnancy (less than 30 years)
B) Early menarche (less than 12 years)
C) Early menopause (less than 55 years)
D) Multiple episodes of breast-feeding
E) Remote oral contraceptive use

A

B) Early menarche (less than 12 years)

Early menarche is the highest risk factor for breast cancer of the options listed. Late first pregnancy, late menopause, no breast-feeding, and recent oral contraceptive use are also risk factors for breast cancer but are not as high risk.

138
Q
A 42-year-old woman with a 3-cm invasive ductal carcinoma of the right breast is evaluated for breast reconstruction. She has not decided how she wants to manage her contralateral breast. Regarding eliciting a family history, which of the following cancers is associated with a mutation in a breast cancer-susceptibility gene?
A) Colon
B) Esophageal
C) Lung
D) Pancreatic
E) Thyroid
A

D) Pancreatic

The breast cancer-susceptibility gene types 1 and 2 (BRCA1 and BRCA2) are tumor suppressor genes. Mutations in BRCA1 and BRCA2 are associated with hereditary breast and ovarian cancers. Additionally, they can be associated with increased risks of pancreatic and prostate cancer. Thyroid, lung, esophageal, and colon cancer are not associated with increased risks of BRCA1 and BRCA2 mutations.

139
Q
A 35-year-old woman comes to the office for consultation regarding prophylactic mastectomy and breast reconstruction. The patient’s mother and sister were diagnosed with bilateral breast cancer in their premenopausal years. Genetic testing for BRCA mutations is negative. Which of the following best estimates this patient’s lifetime risk of breast cancer?
A) 5%
B) 13%
C) 20%
D) 45%
E) 80%
A

D) 45%

The cumulative lifetime risk for a 35-year-old woman whose mother and sister had breast cancer is estimated to be approximately 15%. The risk may increase to as high as 45% if those cancers were premenopausal and bilateral. BRCA hereditary cancer is characterized by autosomal dominant genetics with multiple family members in each generation being affected. For patients with BRCA1 mutation, the risk of breast cancer has been estimated to be between 50 and 80% by age 65 years. The risk of developing ovarian cancer has been estimated to be 10% by age 60 years.

140
Q

A 39-year-old woman with a history of fibrocystic breast lesions comes to the office for consultation. She has no family history of breast cancer. Results of routine mammograms have been negative; she has never undergone biopsy. Which of the following is the most appropriate recommendation for this patient regarding managing her risk of breast cancer?
A) Continue to schedule routine mammograms
B) Refer for mastectomy
C) Schedule core needle biopsy
D) Schedule fine-needle aspiration
E) Schedule genetic testing

A

A) Continue to schedule routine mammograms

Studies have shown the fibrocystic changes alone in the breast are not directly linked to an increased risk of breast cancer, so there is no indication for fine-needle aspiration or core biopsy. Cancer risk increases in benign breast disease with increased proliferation and atypical hyperplasia. Even with no family history of breast cancer, it is recommended that the patient continue routine mammograms. Fibrocystic breast disease has not been linked in the literature to an increased risk of mutations of the BRCA genes; therefore, genetic testing is unnecessary.

141
Q
An otherwise healthy 37-year-old woman presents for delayed microsurgical breast reconstruction. Which of the following is associated with use of tamoxifen?
A) Hemodynamic instability
B) Impaired wound healing
C) Increased bleeding
D) Seroma formation
E) Thromboembolic events
A

E) Thromboembolic events

Breast cancers that are estrogen receptor positive may be responsive to adjuvant chemotherapy with selective estrogen receptor modulators such as tamoxifen, which can reduce recurrence and mortality. Tamoxifen is associated with thromboembolic events, such as deep venous thrombosis and pulmonary embolism. This prothrombotic effect has been postulated to be secondary to the effect of tamoxifen on estrogen receptors that are abundant within vascular endothelium.

Tamoxifen has been shown to be associated with increased rates of total flap loss and decreased rates of flap salvage when taken within 28 days of microsurgical breast reconstruction, which represents two half-lives of the active metabolite of tamoxifen (N-desmethyl tamoxifen, t1/2=14 days). It has therefore been recommended that in patients undergoing microsurgical breast reconstruction, tamoxifen be held for at least 28 days preoperatively. Some authors have further advised holding the medication postoperatively in addition to preoperatively.

Tamoxifen is not associated with impaired wound healing, increased bleeding, hemodynamic instability, or seroma formation.

142
Q

An otherwise healthy 38-year-old woman undergoes prophylactic bilateral mastectomy and immediate reconstruction with deep inferior epigastric artery perforator (DIEP) free flaps. Intraoperatively, the left DIEP flap appears congested before the conclusion of the case. The left deep inferior epigastric artery and vein (DIEA and DIEV) were anastomosed to the proximal internal mammary vessels. The vascular pedicle is evaluated and each anastomosis appears patent and not kinked; however, the venous congestion persists. Which of the following is the most appropriate management?
A) Anastomose the superficial inferior epigastric vein to an internal mammary vessel perforator
B) Convert to left prosthetic reconstruction
C) Infuse tissue plasminogen activator (tPA) to the DIEA
D) Initiate leech therapy
E) Revise the DIEV anastomosis to the retrograde internal mammary vessel limb

A

A) Anastomose the superficial inferior epigastric vein to an internal mammary vessel perforator

Venous drainage of the lower abdominal skin and subcutaneous tissue occurs primarily through the superficial venous system and secondarily through the deep venous system, with perforating veins interconnecting the two systems. These communicating veins have been identified on computed tomography angiography in approximately 90% of abdominal walls in vivo. The majority of the remaining 10% of patients likely have communicating veins that are too small to visualize or are absent. In these cases of anatomical superficial venous system dominance, venous drainage is dependent on the superficial venous system. A recently published 2012 article by Sbitany et al. demonstrated that the incidence of intraoperative venous congestion secondary to persistent superficial venous system dominance was 0.9% in 1201 muscle-sparing transverse rectus abdominis musculocutaneous and deep inferior epigastric artery perforator free flaps. A free flap that becomes congested after reperfusion in the operating room should be assessed immediately for possible etiologies including twisting, kinking, tension, or vasospasm of the vascular pedicle. If a technical problem is ruled out and the venous anastomosis remains patent, obligatory enhancement of venous drainage with the superficial venous system is necessary to salvage the free flap rather than revision of the original anastamosis. Various methods include an anastomosis of the superficial inferior epigastric vein (SIEV) to the DIEV system or any chest wall vein, including the retrograde limb of the internal mammary vessel, the branch of the internal mammary vessel, or the thoracodorsal system. This requires preemptive planning and sparing of the superficial epigastric vein or SIEV during the dissection of the flap. A vein graft can be utilized if additional length is necessary. Another option is to substitute the DIEV anastomosis with the SIEV.

Tissue plasminogen activator (tPA) would not be indicated in this scenario, as it is used as a thrombolytic and there is no evidence of vascular thrombosis. Revising the DIEV anastomosis would be moot because it is patent and the deep system is being drained. Leech therapy is useful for venous congestion, but primarily as an adjunct after potential surgical etiologies have been addressed. Sacrificing the free flap without first attempting salvage is not warranted, and using a prosthetic would be possible only if prior patient consent were obtained.

143
Q

A 60-year-old woman with breast cancer undergoes a transverse rectus abdominis musculocutaneous flap breast reconstruction after mastectomy. She has no allergies. Weight is 200 lb (91 kg). Estimated blood loss is 200 mL. Duration of the operation is 3 hours and 50 minutes. Administration of cefazolin before skin incision is planned as prophylaxis against surgical site infection. Which of the following is the most appropriate dosage and timing of this injection?
Dose Timing Redosing
A) 1g 5 minutes prior after 150 mL of blood loss
B) 1g 15 minutes prior no
C) 1g 40 minutes prior no
D) 2g 15 minutes prior no
E) 2g 40 minutes prior no

A

E) 2g 40 minutes prior no

There has been a renewed interest in perioperative antibiotics in recent years toward more appropriate use to decrease surgical site infection (SSI) while decreasing the incidence of resistant bacteria.

Current recommendations are to administer a single perioperative dose of antibiotics against common skin flora (gram positive), usually using a first-generation cephalosporin. However, the following recommendations may be underappreciated:

Cefazolin intravenous: 1 g if 80kg
Alternatives: clindamycin 600 to 900mg intravenously;
vancomycin 1 to 1.5g intravenously
In this case, the patient weighs 200lb (91kg), so 2g of cefazolin is the recommended dosage.

Additionally, an important factor is the timing of the administration of the perioperative antibiotics in order to achieve proper skin levels before incision. In one study by Classen et al., published in the New England Journal of Medicine in 1992, the optimal time was between 2 hours before the operation and skin incision, as greater than 2 hours before and any time after skin incision led to marked increases in the relative risk of SSIs. A follow-up by Weber et al. in 2008 narrowed the most appropriate window to between 30 and 59 minutes before skin incision.

Finally, there is the issue of redosing. Current recommendations are to redose if there is excessive blood loss (>1500 mL) or if there are long procedures where one exceeds the half-life of the antibiotic used.

In the clinical scenario described, the most appropriate choice is 2 g of cefazolin, because the patient’s weight is above 80 kg, administered between 30 to 59 minutes before skin incision. Redosing on the basis of blood loss is unnecessary, although one could consider redosing at approximately 4 hours on the basis of half-life.

144
Q
A 42-year-old woman who underwent mastectomy of the right breast 6 months ago is evaluated for delayed autologous breast reconstruction with free tissue transfer. Which of the following medications should be discontinued preoperatively if she is routinely taking it?
A) Diltiazem
B) Fluoxetine
C) Metoprolol
D) Multivitamin
E) Tamoxifen
A

E) Tamoxifen

It is well known that tamoxifen can increase the risk of thrombembolic events. In a retrospective study at MD Anderson Cancer Center, it was shown that patients who received the drug close to the procedure had a significantly higher rate of complications. It is recommended that the patient stop tamoxifen at least 28 days before surgery.

Other listed medications do not have a direct effect on thrombotic complications.

145
Q

When performing immediate breast reconstruction, it is important to reconstruct the lateral inframammary fold. This is because the oncologic extirpation of the breast is carried out to which of the following anatomic landmarks?
A) Anterior edge of the latissimus dorsi muscle
B) Anterior edge of the serratus muscle
C) Lateral edge of the pectoralis major muscle
D) Lateral edge of the pectoralis minor muscle
E) Posterior edge of the serratus muscle

A

A) Anterior edge of the latissimus dorsi muscle

For modified radical and simple mastectomies, the landmarks of dissection are: superiorly to the clavicle, medially to the sternum, inferiorly to the inframammary fold, and laterally to the border of the latissimus dorsi muscle. The pectoralis major muscle fascia is resected with the specimen.

The recreation of the inframammary fold is important for shaping in breast reconstruction and care must be taken to evaluate and repair both the inferior and lateral components of the inframammary fold.

146
Q

A 52-year-old woman is evaluated for breast reconstruction after modified radical mastectomy with adjuvant chemotherapy and radiation therapy 18 months ago. Her last radiation treatment was 8 months ago. BMI is 29 kg/m2. Examination today shows hyperpigmentation of the right chest wall with no redundancy of the mastectomy skin flaps. Her contralateral breast is a D cup with grade III ptosis. Which of the following methods will create the best symmetry for this patient?
A) Deep inferior epigastric perforator flap
B) Gel breast implant and acellular dermal matrix
C) Latissimus dorsi musculocutaneous flap
D) Tissue expander and acellular dermal matrix placement with planned staged exchange for permanent gel implant
E) Tissue expander placement alone with planned staged exchange for permanent gel implant

A

A) Deep inferior epigastric perforator flap

The deep inferior epigastric perforator flap would give the patient autologous tissue reconstruction with ample tissue for skin resurfacing and soft tissue for volume. In this radiated patient with a tight skin envelope, a tissue expander/implant, with or without acellular dermal matrix, would be difficult to create an appropriately ample skin envelope and the patient would be at higher risk for wound-healing problems and capsular contracture. The latissimus dorsi musculocutaneous flap, although an autologous tissue reconstruction, would have insufficient volume to adequately match this patient’s contralateral side. It would have to be combined with an implant.

147
Q

An otherwise healthy 67-year-old woman with advanced breast cancer is scheduled to undergo mastectomy and immediate reconstruction with a free transverse rectus abdominis musculocutaneous (TRAM) flap. BMI is 35 kg/m2. Which of the following is most appropriate for deep venous thrombosis prophylaxis?
A) Aspirin and intermittent pneumatic compression stockings
B) Elastic compression stockings only
C) Intermittent pneumatic compression stockings only
D) Low-molecular-weight heparin and intermittent pneumatic compression stockings
E) Positioning and early ambulation

A

D) Low-molecular-weight heparin and intermittent pneumatic compression stockings

This patient is at highest risk for deep venous thrombosis and she will require combination therapy of compression stockings and chemical prophylaxis.

Perhaps the most well-regarded set of guidelines in this matter comes from the American College of Chest Physicians. These guidelines provide treatment recommendations based on a patient’s risk classification. Caprini model offers a very user-friendly method of calculating patient risk factors and categorizing them into low, moderate, high, and highest risks.

The patient in question has risk factors (age, malignancy, and major surgery) that would put her in the highest risk category. In the highest risk category, prophylaxis will require combination therapy or warfarin.

148
Q

A 48-year-old woman undergoes radical resection of the left breast, left hemisternectomy, four-rib resection, and visceral pleurectomy because of invasive inflammatory breast cancer extending through the thoracic wall into the mediastinum and chest. Examination of specimens obtained from intraoperative frozen section biopsies shows no residual disease. Postoperative adjuvant chemotherapy and radiation therapy to the area are planned. An intraoperative photograph of the 17 × 17-cm defect is shown. Which of the following is the most appropriate technique for soft-tissue reconstruction?
A) Left latissimus muscle flap with skin graft
B) Left rectus muscle turnover flap and skin graft
C) Omental flap and skin graft
D) Reverse abdominoplasty flap
E) Right transverse rectus abdominis musculocutaneous (TRAM) flap

A

E) Right transverse rectus abdominis musculocutaneous (TRAM) flap

The most appropriate option for this patient would be a right TRAM flap. The defect encompasses a composite defect of the left side of the sternum, including the left internal mammary artery, four contiguous ribs, and the entire left breast. The question specifically asks for soft-tissue thoracic reconstruction. The analysis of the defect yields a very large soft-tissue requirement. Given this, the most logical choice of flaps to provide this amount of soft tissue on a reliable pedicle would be a right (contralateral) TRAM flap. Furthermore, this flap could also be shaped to provide a breast mound versus all the other choices.

A left latissimus muscle flap with skin graft is not the optimal choice as it would not provide sufficient soft-tissue coverage with its associated skin paddle without undue donor site morbidity in this particular patient with a large defect that spans parasternal and lateral chest wall. As a muscle-only flap with skin graft, however, it may be considered as a backup option in case of primary flap failure.

An omental flap with skin graft is an option for reconstruction of this defect; however, it would not be the most appropriate option in this case as it would necessitate an otherwise unnecessary intra-abdominal procedure and would not reshape a breast mound. It has been shown, however, to be optimal in contaminated areas due to its associated lymphatic vessels and nodes associated with its pedicle, so in different circumstances, this may be a preferred choice.

A left rectus turnover flap with skin graft would not be a good option for two reasons: the left internal mammary artery has been harvested, thereby compromising the superior epigastric vessel on which this flap would be based; and it would not cover the entirety of the defect.

A reverse abdominoplasty flap would not be able to reach the full superior extent of the defect.

149
Q

A 45-year-old woman comes to the office for consultation regarding mastectomy and immediate breast reconstruction because of recurrent right-sided breast cancer. She underwent breast-conserving therapy and radiation therapy 5 years ago. BMI is 23 kg/m2. The patient is otherwise healthy, and she does not smoke cigarettes. On examination, the breast is soft with obvious fibrosis. Use of which of the following has the highest risk for reconstructive failure in this patient?
A) Deep inferior epigastric artery perforator (DIEP) flap
B) Free transverse rectus abdominis musculocutaneous (TRAM) flap
C) Latissimus flap with prosthesis
D) Pedicled TRAM flap
E) Tissue expander and prosthesis

A

E) Tissue expander and prosthesis

Pre-reconstruction radiation is typically seen in two groups of patients: those who underwent mastectomy without reconstruction followed by radiation therapy, or those who underwent breast-conserving therapy and radiation with recurrence or new cancer. Although these patients may present with a reasonable skin envelope, complication rates associated with tissue expander/prosthesis have been reported as high as 70%, with a 40% rate of failure or conversion to flap. Additionally, aesthetic outcomes in most patients who completed expander/prosthesis reconstruction after previous radiation therapy were deemed good or very good compared with the majority of non-irradiated patients who achieved excellent results. Tissue expander/prosthesis can be considered as an option for patients with a history of previous irradiation who wish to avoid the scars and recovery of flap-type reconstructions; however, they should be counseled of the high risks of complications.

150
Q

A 74-year-old woman comes to the office because of a 2-month history of increasing soreness over the left lateral chest wall. History includes bilateral radical mastectomy and adjuvant radiation to the left chest 30 years ago because of left-sided breast cancer. On examination, a small, nonhealing 1 × 2-cm wound is visible in the subaxillary region within an area of dense fibrotic skin. A photograph is shown. Which of the following is the most appropriate next step?
A) Biopsy of the nonhealing wound
B) Coverage with a left latissimus muscle flap and skin grafting
C) LHyperbaric oxygen treatment
D) MRI of the chest wall
E) Oral administration of antibiotics for 7 days

A

A) Biopsy of the non healing wound

This patient’s presentation is consistent with late radiation tissue injury (LRTI), which occurs in 5 to 15% of long-term cancer survivors who have received radiation and can occur months to years (even decades) after treatment. It can vary significantly with age, dose, and site of treatment. Characteristics include progressive tissue deterioration secondary to decreased vascularity followed by replacement of normal soft-tissue architecture by dense fibrotic tissue until there is insufficient oxygen delivery to sustain normal function. When LTRI of the chest wall progresses to skin breakdown and ulceration, as has happened in this patient, a biopsy is always required to rule out recurrence of the primary tumor or a radiation-induced squamous cell carcinoma or soft-tissue sarcoma.

Hyperbaric oxygen therapy has been proposed as a treatment modality that can improve tissue quality and prevent tissue breakdown in irradiated areas. While most data on this relate to treatment of osteoradionecrosis of the mandible, there have been some reports of its use on the chest wall after irradiation due to breast cancer. However, the question specifically asks for the most appropriate next step, which would be biopsy rather than hyperbaric oxygen therapy.

The principles of management once malignancy has been ruled out include debridement of necrotic tissues (including ribs) and reconstruction with well-vascularized flaps. In this case, a left latissimus muscle flap and skin graft was used for reconstruction after aggressive debridement. No alloplastic material was placed or thoracic cage reconstruction performed. This is common in these types of patients as excessive fibrosis caused by the radiation to the chest wall prevents loss of respiratory efficiency through paradoxical motion.

CT scan or MRI may be useful in evaluating the size, extent, and nature of the problem, but does not take priority over tissue biopsy.

The redness that is visualized on the patient’s skin in the case is related to post-radiation changes and telangiectasias, which are common in LTRI, and is not infection. Therefore, antibiotics would be unnecessary.

151
Q

A 34-year-old woman is scheduled to undergo bilateral prophylactic mastectomy because of a strong family history of breast cancer. Reconstruction with free flaps from the upper inner thigh is planned. Which of the following is the pedicle to these flaps?
A) Ascending branch of the lateral femoral circumflex artery
B) Descending branch of the lateral femoral circumflex artery
C) Medial femoral circumflex artery
D) Superficial circumflex iliac artery
E) Superficial femoral artery

A

C) Medial femoral circumflex artery

The transverse upper gracilis (TUG) flap is a free flap that can be used in breast reconstruction. It is particularly useful in cases where the abdominal donor site is not available or not desired. The flap incorporates the skin and subcutaneous tissues of the upper inner thigh, in the region of the medial thigh lift. The TUG flap is based on the gracilis vessels and perforators from the descending branch of the medial femoral circumflex artery, which forms the dominant pedicle. This arises from the profunda femoris artery. Modifications of this flap have been proposed to increase the volume available for free tissue transfer.

The ascending branch of the lateral femoral circumflex artery provides blood supply to the tensor fascia lata flap.

The descending branch of the lateral femoral circumflex artery forms the pedicle for the anterolateral thigh flap.

The superficial circumflex iliac artery forms the pedicle for the groin flap.

Branches from the superficial femoral artery provide minor pedicles to the distal portion of the gracilis muscle, but are less important and distal to the area of the TUG flap.

152
Q
A 41-year-old woman comes to the office because of an invasive ductal carcinoma of the left breast. On mammography, the tumor is 3 cm from the nipple and measures 4 cm. A left-sided lateral periareolar scar extending from the 12 o'clock to the 3 o'clock position from a previous biopsy is noted. The patient wishes to undergo a nipple-sparing mastectomy. Which of the following findings places the patient at greatest oncologic risk, including risk for de novo or recurrent cancer or inadequate surgical margins, with this procedure?
A) Distance of tumor to nipple
B) Patient age
C) Presence of the periareolar scar
D) Size of tumor
E) Type of tumor
A

D) Size of tumor

As surgical approaches to breast cancer treatment have evolved, nipple-sparing mastectomy (NSM) has emerged as an alternative to other approaches. It was initially used for prophylactic mastectomies, and patients reported increased satisfaction and body image with nipple-areola complex (NAC) preservation. The role of NSM has been expanded to therapeutic mastectomy, and with that there has been increased research in the oncologic safety of this approach.

Studies have evaluated therapeutic NSM in the context of invasive ductal carcinoma, invasive lobular carcinoma, and ductal carcinoma in situ. The type of cancer does not appear to be associated with the oncologic safety of NSM. Several studies have demonstrated an inverse association between NAC involvement and distance of the tumor from the nipple. While these studies have varied in their distance cutoffs, nipple involvement is reported to be over 50% when the tumor-nipple distance is less than 2 cm, as noted in one study. There is a direct correlation between tumor size and NAC involvement—the same study cited data that when the tumor was greater than 4 cm, the likelihood of nipple involvement was greater than 50%.

One published screening algorithm for plastic surgery includes tumor size less than 3 cm, and tumor location greater than 2 cm from the nipple as criteria for NSM candidacy.

A periareolar scar, if large, may compromise the blood supply to the NAC. Acceptable incisions for NSM, however, include a periareolar incision of 25 to 50%.

153
Q
A 26-year-old woman with a strong family history of breast cancer undergoes genetic testing. She is found to have a deleterious mutation of the BRCA1 gene. Which of the following best describes her lifetime risk for the development of breast cancer when compared with women without this mutation?
A) Two times greater
B) Six times greater
C) Ten times greater
D) Fifteen times greater
E) Twenty times greater
A

B) Six times greater

A woman’s lifetime risk of developing breast and/or ovarian cancer is greatly increased if she inherits an altered BRCA1 or BRCA2 gene. Women with an inherited alteration in one of these genes have an increased risk of developing these cancers at a young age (before menopause) and often have multiple close family members with the disease. These women may also have an increased chance of developing colon cancer.

Men with an altered BRCA1 or BRCA2 gene also have an increased risk of breast cancer (primarily if the alteration is in BRCA2) and possibly prostate cancer. Alterations in the BRCA2 gene have also been associated with an increased risk of lymphoma, melanoma, and cancers of the pancreas, gallbladder, bile duct, and stomach in some men and women.

According to estimates of lifetime risk, approximately 13.2% (132 of 1000 individuals) of women in the general population will develop breast cancer, compared with estimates of 36 to 85% (360 to 850 of 1000) of women with an altered BRCA1 or BRCA2 gene. In other words, women with an altered BRCA1 or BRCA2 gene are 3 to 7 times more likely to develop breast cancer than women without alterations in those genes. Lifetime risk estimates of ovarian cancer for women in the general population indicate that 1.7% (17 of 1000) will get ovarian cancer, compared with 16 to 60% (160 to 600 of 1000) of women with altered BRCA1 or BRCA2 genes. No data are available from long-term studies of the general population comparing the cancer risk in women who have a BRCA1 or BRCA2 alteration with women who do not have an alteration in these genes. Therefore, these figures are estimated ranges that may change as more research data are added.

154
Q

A 42-year-old woman comes to the office because she is dissatisfied with an obvious step-off between the chest wall and the superior pole of the breast 6 months after she underwent immediate expander reconstruction of the left breast. No further adjuvant therapy was indicated. After full expansion, the tissue expander was removed, and a permanent smooth, round silicone prosthesis was placed. BMI today is 28 kg/m2. Examination shows a well-defined inframammary fold. The volume is matched to the contralateral breast. Which of the following is the most appropriate surgical procedure for correction of this patient’s deformity?
A) Exchange of the silicone prosthesis with a silicone prosthesis of a larger volume and dimension
B) Exchange of the silicone prosthesis with a saline prosthesis of the same volume and dimension
C) Fat grafting to the superior pole and chest wall
D) Placement of acellular dermal matrix as an inferolateral sling
E) Removal of the prosthesis and reconstruction with a deep inferior epigastric perforator flap

A

C) Fat grafting to the superior pole and chest wall

Fat grafting has been shown to successfully address acquired contour deformities in breast reconstruction. Changing the prosthesis to a saline prosthesis of the same size and dimension will not address the problem of the step-off between the prosthesis and the chest wall in this slender patient (BMI 20 kg/m2) which is due to decreased soft-tissue coverage in the superior pole. Likewise, increasing the size and dimensions of the silicone prosthesis would not correct this contour deformity and would also lose the symmetry with the contralateral breast. This patient, without excess lower abdominal skin and subcutaneous fat, is not a candidate for a deep inferior epigastric perforator flap. Placement of acellular dermal matrix as an inferolateral sling is typically performed in the initial stage of tissue expansion and reconstruction. It can help define the inframammary fold, which this patient does not need

155
Q
A 33-year-old woman comes to the office with a 6-cm rapidly growing tumor of the left breast. She wears a size 36C brassiere. The tumor has a bluish hue and skin veins are dilated. A phyllodes tumor is diagnosed, and surgical excision is planned. Which of the following is the most appropriate surgical procedure to treat this patient?
A) Excision with 1-cm margin
B) Excision with 2-cm margin
C) Excisional biopsy
D) Modified radical mastectomy
E) Radical mastectomy
A

A) Excision with 1-cm margin

Phyllodes are large benign tumors that occur primarily in the perimenopausal patient. Previously, they were referred to as cystosarcoma phyllodes, a term coined in 1838 because the tumors are fleshy and have a gross leaf-like intracanalicular growth pattern. However, this is a misnomer because these tumors do not behave like sarcomas and are rarely malignant. The histologic characteristics that separate fibroadenomas from phyllodes tumors are not well defined and have been somewhat controversial. Nevertheless, phyllodes tumors typically are large fibroadenomas that histologically have more stromal cellularity than that seen in the typical fibroadenoma. The classification of benign versus malignant phyllodes tumors is not sharply delineated, and the term borderline lesion may be more appropriate. Borderline lesions have more mitoses per high-power field and moderate nuclear pleomorphism. They have a tendency to recur after local excision but do not demonstrate true malignant behavior. When metastases of a phyllodes tumor have been reported, there have been obvious sarcomatous elements such as liposarcoma or rhabdomyosarcoma in the lesion.

The surgical treatment of phyllodes tumors has recently been redefined. In the past, simple or radical mastectomies were recommended for the treatment of large phyllodes tumors. Currently, most surgeons perform more conservative surgery. Several clinical studies have recommended the excision of tumors with 1-cm clear margins or mastectomy if breast conservation is impossible.

156
Q
A 41-year-old man is referred to the office because of a mass on his left breast that has been growing rapidly for 1 month. Examination of the specimen obtained on core needle biopsy confirms invasive mammary carcinoma. Genetic testing results are positive for BRCA2. In addition to an increased risk of male breast cancer, which of the following other types of cancer is most likely to be associated with this patient?
A) Colon
B) Lung
C) Pancreatic
D) Renal
E) Thyroid
A

C) Pancreatic

Men with harmful BRCA1 mutations also have an increased risk of breast cancer and, possibly, pancreatic cancer, testicular cancer, and early onset prostate cancer. However, male breast cancer, pancreatic cancer, and prostate cancer appear to be more strongly associated with BRCA2 gene mutations. Colon, lung, renal, and thyroid cancers are not associated with the BRCA1 and BRCA2 gene mutations.

157
Q
A 40-year-old woman is referred for reconstruction following mastectomy for a peripherally located ductal carcinoma in situ. A nipple-sparing mastectomy with immediate, single-stage prosthesis reconstruction with acellular dermal matrix is planned. Which of the following interventions is most appropriate to ensure that no residual cancer exists?
A) Chemotherapy
B) Ductal washing
C) Radiation therapy
D) Retroareolar frozen section
E) Sentinel lymph node evaluation
A

D) Retroareolar frozen section

More plastic surgeons are performing reconstruction for women pursuing prophylactic mastectomy, which is requested quite frequently to avoid cancer recurrence and to achieve optimal aesthetic outcome. Exclusion criteria for nipple-sparing mastectomy include tumors greater than 3 cm, clinical invasion of the nipple-areola complex, tumors within 2 cm of the nipple, evidence of multicentric disease, positive intraoperative retroareolar frozen section, or nodal disease. If carcinoma is found in the retroareolar tissue, the nipple-areola complex must be removed. A patient who would require sentinel lymph node evaluation, radiation therapy, or chemotherapy would not be an appropriate candidate for nipple-sparing mastectomy. Annual mammography is recommended for any patient with a history of breast cancer and is not specific to the issue of nipple-sparing mastectomy. Ductal washing is not relevant for this pathology.